Một số vấn đề về bất đẳng thức bậc bốn ba biến

pdf 23 trang Người đăng khoa-nguyen Lượt xem 1643Lượt tải 0 Download
Bạn đang xem 20 trang mẫu của tài liệu "Một số vấn đề về bất đẳng thức bậc bốn ba biến", để tải tài liệu gốc về máy bạn click vào nút DOWNLOAD ở trên
Một số vấn đề về bất đẳng thức bậc bốn ba biến
MỘT SỐ VẤN ĐỀ VỀ BẤT ĐẲNG THỨC
BẬC BỐN BA BIẾN
Nguyễn Văn Huyện1
Tống Hữu Nhân2
1. Mở đầu
Trên tạp chí IJPAM 3 nhóm tác giả Vasile Cirtoaje 4 và Võ Quốc Bá Cẩn 5 có đề xuất
định lý:
Với a, b, c là ba số thực, xét đa thức
F (a, b, c) =
∑
a4 + A
∑
b2c2 +Babc
∑
a+ C
∑
b3c+D
∑
bc3.
Nếu 1 + A+B + C +D = 0, bất đẳng thức F (a, b, c) > 0 sẽ đúng khi và chỉ khi
3(1 + A) > C2 + CD +D2.
Với định lý này lớp các bài toán bậc 4 ba biến số đã được giải quyết triệt để. Tuy nhiên
việc chứng minh và áp dụng nó trong các kỳ thi không phải là điều dễ dàng. Theo quan
điểm cá nhân định lý này giống như một tiêu chuẩn trong việc sáng tạo ra các bất đẳng
thức mới hơn là một phương pháp để giải quyết các bài toán. Điều mà chúng ta quan
tâm không chỉ là những định lý tổng quát mà còn là những công cụ không quá quá phức
1Sinh viên trường Đại học Giao Thông Vận Tải thành phố Hồ Chí Minh.
2Sinh viên trường Đại học Y Dược thành phố Hồ Chí Minh.
3International Journal of Pure and Applied Mathematics
4Trường đại học Department of Automatic Control and Computers, Ploiesti, Romania
5Hà Nội
1
Một số vấn đề về bất đẳng thức bậc bốn ba biến
tạp để sử dụng trong phòng thi. Vì thế, trong bài viết nhỏ này, chúng tôi xin chia sẻ một
vài kỹ thuật nhỏ khá hiệu quả trong việc xử lý các bất đẳng thức bậc 4 ba biến.
2. Các bài toán đối xứng
Năm 2011 trên diễn đàn toán học artofproblemsolving.com thành viên mudok có đề xuất
bài toán sau đây:
Bài 2.1. Cho các số thực a, b, c thỏa mãn a+ b+ c = 0. Chứng minh rằng
(ab+ bc+ ca)2 + 9abc > 3(ab+ bc+ ca). (2.1.1)
Lời giải. Cách tự nhiên nhất là ta sẽ rút c = −a− b rồi thay vào (2.1.1), và được[
ab− (a+ b)2]2 − 9ab(a+ b) > 3 [ab− (a+ b)2] ,
(a2 + ab+ b2)2 + 3(a2 + ab+ b2) > 9ab(a+ b).
Quan sát một chút ta thấy bất đẳng thức lúc này chứa ba đại lượng a2 +ab+ b2, ab, a+ b
và chúng có mối liên hệ với nhau thông qua đánh giá
a2 + ab+ b2 > 3
4
(a+ b)2 > 3ab,
suy ra
(a2 + ab+ b2)2 + 3(a2 + ab+ b2) > 9
16
(a+ b)4 + 9ab.
Như vậy ta chỉ cần chứng minh
(a+ b)4
16
+ ab > 9ab(a+ b).
Rất tiếc bất đẳng thức này không đúng (có thể kiểm tra với a = −1, b = 1). Do đó ta sẽ
đổi lại đánh giá theo kiểu
(a2 + ab+ b2)2 + 3(a2 + ab+ b2) > (3ab)2 + 3 · 3
4
(a+ b)2
= 9a2b2 +
9
4
(a+ b)2,
(2.1.2)
và quy bài toán về chứng minh
a2b2 +
(a+ b)2
4
> ab(a+ b).
Bất đẳng thức này đúng theo bất đẳng thức AM-GM, nhưng có một lỗi trong đánh giá
(2.1.2) đó là (a2 + ab+ b2)2 > (3ab)2 chỉ đúng khi ab > 0, nhưng ta có thể khắc phục lỗi
này bằng kỹ thuật sau.
Do ab · bc · ca = a2b2c2 > 0 nên trong ba số ab, bc, ca sẽ có ít nhất một số không âm, giả
sử ab > 0. Với những lập luận trên, áp dụng bất đẳng thức AM-GM, ta có
a2 + ab+ b2 > 3
4
(a+ b)2 > 3ab > 0. (2.1.3)
2
Một số vấn đề về bất đẳng thức bậc bốn ba biến
Suy ra
(a2 + ab+ b2)2 + 3(a2 + ab+ b2) > 9a2b2 + 9(a+ b)
2
4
.
Ta sẽ chứng minh
a2b2 +
(a+ b)2
4
> ab(a+ b).
Cũng theo bất đẳng thức AM-GM thì
a2b2 +
(a+ b)2
4
> 2
√
a2b2 · (a+ b)
2
4
= ab |a+ b| > ab(a+ b). (2.1.4)
Đẳng thức xảy ra khi (2.1.3) và (2.1.4) trở thành đẳng thức, tức
a = b
a2b2 =
(a+ b)2
4
Giải hệ này ta được a = b = c = 0 và a = b = 1, c = −2. Bài được chứng minh.
Bài 2.2. Cho ba số thực a, b, c thỏa mãn a+ b+ c = 3. Chứng minh rằng
3(a4 + b4 + c4) + a2 + b2 + c2 + 6 > 6(a3 + b3 + c3). (2.2.1)
Lời giải. Đây cũng là một bài toán đối xứng, nên ta sẽ tìm cách đưa điều kiện về dạng
tổng bằng 0, rồi dùng phép thế như trên. Rất tự nhiên đặt a = x+1, b = y+1, c = z+1,
khi đó
x+ y + z = (a− 1) + (b− 1) + (c− 1) = 0.
Với phép đặt này thì
a2 + b2 + c2 = (x+ 1)2 + (y + 1)2 + (z + 1)2
= x2 + y2 + z2 + 2(x+ y + z) + 3
= x2 + y2 + z2 + 3,
tương tự
a3 + b3 + c3 = x3 + y3 + z3 + 3(x2 + y2 + z2) + 3,
a4 + b4 + c4 = x4 + y4 + z4 + 4(x3 + y3 + z3) + 6(x2 + y2 + z2) + 3.
Bất đẳng thức (2.2.1) trở thành
3(x4 + y4 + z4) + 6(x3 + y3 + z3) + x2 + y2 + z2 > 0. (2.2.2)
Giả sử xy > 0, rồi thay z = −x− y vào (2.2.2), ta được
3[x4 + y4 + (x+ y)4] + 6[x3 + y3 − (x+ y)3] + x2 + y2 + (x+ y)2 > 0,
3(x4 + 2x3y + 3x2y2 + 2xy3 + y4) + x2 + xy + y2 > 9xy(x+ y)
3(x2 + xy + y2)2 + x2 + xy + y2 > 9xy(x+ y).
3
Một số vấn đề về bất đẳng thức bậc bốn ba biến
Áp dụng bất đẳng thức AM-GM, ta có
x2 + xy + y2 > 3
4
(x+ y)2 > 3xy > 0, (2.2.3)
suy ra
3(x2 + xy + y2)2 + x2 + xy + y2 > 27x2y2 + 3
4
(x+ y)2.
Do đó ta chỉ cần chứng minh
9x2y2 +
(x+ y)2
4
> 3xy(x+ y).
Cũng theo bất đẳng thức AM-GM, thì
9x2y2 +
(x+ y)2
4
> 2
√
9x2y2 · (x+ y)
2
4
= 3xy |x+ y| > 3xy(x+ y). (2.2.4)
Đẳng thức xảy ra khi (2.2.3) và (2.2.4) trở thành đẳng thức, tức x, y là nghiệm của hệx = y27x2y2 = 3
4
(x+ y)2
Giải hệ này ta được x = y = 0 hoặc x = y = 1
3
, suy ra a = b = c = 1, hoặc a = b = 4
3
, c = 1
3
cùng các hoán vị. Bài toán được chứng minh.
Sau 2 ví dụ đầu ta thấy ý tưởng chính để giải quyết các bất đẳng thức đối xứng bậc 4
được “mô hình” hóa như sau:
(1) Nếu bất đẳng thức có điều kiện a + b + c = 3 thì ta sẽ tiến hành đổi biến sang
x = a − 1, y = b − 1, z = c − 1, còn nếu bất đẳng thức có các điều kiện khác thì
ta cố gắng thuần nhất bài toán hoặc thông qua một số đánh giá để đưa bài toán về
đồng bậc sau đó chuẩn hóa a+ b+ c = 3, rồi sử dụng các biến đổi
a4 + b4 + c4 = x4 + y4 + z4 + 4(x3 + y3 + z3) + 6(x2 + y2 + z2) + 3,
a3 + b3 + c3 = x3 + y3 + z3 + 3(x2 + y2 + z2) + 3,
a2 + b2 + c2 = x2 + y2 + z2 + 3,
ab+ bc+ ca = xy + yz + zx+ 3,
abc = xyz + xy + yz + zx+ 1
· · ·
(2) Thay z = −x − y rồi biến đổi về hai đại lượng x2 + xy + y2, xy (x+ y) thông qua
các đẳng thức
x4 + y4 + z4 = 2(x2 + xy + y2)2,
x3 + y3 + z3 = −3xy(x+ y),
x2 + y2 + z2 = 2(x2 + xy + y2),
xy + yz + zx = −(x2 + xy + y2),
xyz = −xy(x+ y),
· · ·
4
Một số vấn đề về bất đẳng thức bậc bốn ba biến
Cuối cùng ta sẽ thu được bất đẳng thức có dạng
A(x2 + xy + y2)
2
+B(x2 + xy + y2) + Cxy (x+ y) +D > 0.
Từ đó tùy vào các hệ số các A, B, C, D mà ta có những đánh giá thích hợp để giải
quyết bài toán.
Rất vui mừng vì phát hiện thú vị này chúng tôi đã thử áp dụng vào các bài toán khác và
thu được những kết quả sau:
Bài 2.3. Cho ba số thực a, b, c thỏa mãn a+ b+ c = 3. Chứng minh rằng
(ab+ bc+ ca− 3)2 > 27(abc− 1). (2.3.1)
Lời giải. Đặt a = x+ 1, b = y + 1, c = z + 1, thì x+ y + z = 0. Khi đó
ab+ bc+ ca = xy + yz + zx+ 3.
abc = xyz + xy + yz + xz + 1.
Bất đẳng thức (2.3.1) trở thành
(xy + yz + zx)2 > 27(xyz + xy + yz + xz). (2.3.2)
Giả sử xy > 0, thay z = −x− y vào (2.3.2) và thu gọn lại, ta được
[xy − (x+ y)2]2 > 27[−xy(x+ y) + xy − (x+ y)2],
hay là
(x2 + xy + y2)2 + 27(x2 + xy + y2) + 27xy(x+ y) > 0.
Áp dụng bất đẳng thức AM-GM, ta có
x2 + xy + y2 > 3
4
(x+ y)2 > 3xy > 0,
suy ra
(x2 + xy + y2)2 + 27(x2 + xy + y2) > 9x2y2 + 81
4
(x+ y)2.
Ta sẽ chứng minh
x2y2 +
9
4
(x+ y)2 + 3xy(x+ y) > 0,
bất đẳng thức này đúng vì
x2y2 +
9
4
(x+ y)2 + 3xy(x+ y) =
(3x+ 3y + 2xy)2
4
> 0.
Đẳng thức xảy ra khi a = b = c = 1, hoặc a = b = −2, c = 7 cùng các hoán vị. Chứng
minh của ta được hoàn tất.
Bài 2.4. Cho ba số thực a, b, c thỏa mãn a+ b+ c = 3. Chứng minh rằng
3(a4 + b4 + c4) + 33 > 14(a2 + b2 + c2). (2.4.1)
5
Một số vấn đề về bất đẳng thức bậc bốn ba biến
Lời giải. Đặt a = x + 1, b = y + 1, c = z + 1, thì x + y + z = 0. Bất đẳng thức (2.4.1)
được viết lại như sau
3(x4 + y4 + z4) + 12(x3 + y3 + z3) + 4(x2 + y2 + z2) > 0. (2.4.2)
Giả sử xy > 0, thay z = −x− y vào (2.4.2) và thu gọn lại, ta được
3(x4 + 2x3y + 3x2y2 + 2xy3 + y4) + 4(x2 + xy + y2) > 18xy(x+ y),
3(x2 + xy + y2)2 + 4(x2 + xy + y2) > 18xy(x+ y).
Áp dụng bất đẳng thức AM-GM, ta có
x2 + xy + y2 > 3
4
(x+ y)2 > 3xy > 0,
suy ra
3(x2 + xy + y2)2 + 4(x2 + xy + y2) > 27x2y2 + 3(x+ y)2,
lại có
27x2y2 + 3(x+ y)2 − 18xy(x+ y) = 3(x+ y − 3xy)2 > 0.
Đẳng thức xảy ra khi a = b = c = 1 hoặc a = b = 5
3
, c = −1
3
. Bài toán được chứng
minh.
Bài 2.5. Cho ba số thực a, b, c thỏa mãn a+ b+ c = 3. Chứng minh rằng
(ab+ bc+ ca)2 + 9 > 18abc. (2.5.1)
Lời giải. Đặt a = x + 1, b = y + 1, c = z + 1, thì x + y + z = 0. Khi đó bất đẳng thức
(2.5.1) trở thành
(xy + yz + zx+ 3)2 + 9 > 18(1 + xy + yz + zx+ xyz),
hay
(xy + yz + zx)2 > 12(xy + yz + zx) + 18xyz. (2.5.2)
Giả sử xy > 0, thay z = −x− y vào (2.5.2), ta được
(x2 + xy + y2)2 + 12(x2 + xy + y2) + 18xy(x+ y) > 0.
Áp dụng bất đẳng thức AM-GM, ta có
x2 + xy + y2 > 3
4
(x+ y)2 > 3xy > 0,
suy ra
(x2 + xy + y2)2 + 12(x2 + xy + y2) > 9x2y2 + 9(x+ y)2,
như vậy ta cần chỉ ra
x2y2 + (x+ y)2 + 2xy(x+ y) > 0,
tương đương với
(x+ y + xy)2 > 0.
Đẳng thức xảy ra khi a = b = c = 1 hoặc a = b = −1, c = 5. Bài toán được chứng
minh.
6
Một số vấn đề về bất đẳng thức bậc bốn ba biến
Bài 2.6. Chứng minh rằng bất đẳng thức
(a2 + b2 + c2 − 2ab− 2bc− 2ca)2 + 9(ab+ bc+ ca)2 > 30abc(a+ b+ c), (2.6.1)
luôn đúng với mọi số thực dương a, b, c.
Lời giải. Nếu a+ b+ c = 0 thì (2.6.1) hiển nhiên đúng. Nếu a+ b+ c 6= 0, thay (a, b, c)
bởi (−a, −b, −c) thì bất đẳng thức vẫn không thay đổi nên ta có thể giả sử a+ b+ c > 0,
và chuẩn hóa cho a+ b+ c = 3. Đặt a = x+ 1, b = y+ 1, c = z+ 1, khi đó x+ y+ z = 0.
Bất đẳng thức (2.6.1) trở thành
[4(xy + yz + zx) + 3]2 + 9(xy + yz + zx+ 3)2 > 90(xy + yz + zx+ xyz),
tương đương với
25(xy + yz + zx)2 > 12(xy + yz + zx) + 90xyz. (2.6.2)
Giả sử xy > 0, thay z = −x− y vào (2.6.2) và thu gọn lại, ta được
25(x2 + xy + y2)2 + 12(x2 + xy + y2) + 90xy(x+ y) > 0.
Áp dụng bất đẳng thức AM-GM, ta có
x2 + xy + y2 > 3
4
(x+ y)2 > 3xy > 0,
suy ra
25(x2 + xy + y2)2 + 12(x2 + xy + y2) > 225x2y2 + 9(x+ y)2.
Ta sẽ chứng minh
25x2y2 + (x+ y)2 + 10xy(x+ y) > 0,
điều này đúng vì
25x2y2 + (x+ y)2 + 10xy(x+ y) = (x+ y + 5xy)2 > 0.
Đẳng thức xảy ra khi a = b = c hoặc a = 3b = 3c cùng các hoán vị, như vậy chứng minh
của ta được hoàn tất.
Bài 2.7. Cho ba số thực a, b, c không âm. Chứng minh rằng
3(a2 + b2 + c2) > (a+ b+ c)
(√
ab+
√
bc+
√
ca
)
+
∑
(b− c)2 > (a+ b+ c)2.
(Việt Nam MO 2005)
Lời giải. Thay (a, b, c) bởi (a2, b2, c2) bất đẳng thức cần chứng minh trở thành
3(a4 + b4 + c4) > (a2 + b2 + c2)(ab+ bc+ ca) +
∑
(b2 − c2)2 > (a2 + b2 + c2)2.
Ta chứng minh vế trái
3(a4 + b4 + c4) > (a2 + b2 + c2)(ab+ bc+ ca) + 2(a4 + b4 + c4 − a2b2 − b2c2 − c2a2).
7
Một số vấn đề về bất đẳng thức bậc bốn ba biến
Điều này tương đương với
a4 + b4 + c4 + 2(a2b2 + b2c2 + c2a2) > (a2 + b2 + c2)(ab+ bc+ ca),
(a2 + b2 + c2)2 > (a2 + b2 + c2)(ab+ bc+ ca). (2.7.1)
Chuẩn hóa a + b + c = 3 và đặt a = x + 1, b = y + 1, c = −x− y + 1 vào (2.7.1) và thu
gọn lại, ta được
6(x4 + 2x3y + 3x2y2 + 2xy3 + y4) + 9(x2 + xy + y2) > 0,
6(x2 + xy + y2)2 + 9(x2 + xy + y2) > 0.
Bất đẳng thức này hiển nhiên đúng. Đẳng thức xảy ra khi a = b = c, vế trái được chứng
minh. Tiếp đến ta chứng minh vế bên phải
(a2 + b2 + c2)(ab+ bc+ ca) + 2(a4 + b4 + c4 − a2b2 − b2c2 − c2a2) > (a2 + b2 + c2)2,
bất đẳng thức này tương đương
a4 + b4 + c4 + (a2 + b2 + c2)(ab+ bc+ ca) > 4(a2b2 + b2c2 + c2a2),
a4 + b4 + c4 +abc(a+ b+ c) +ab(a2 + b2) + ca(c2 +a2) + ca(c2 +a2) > 4(a2b2 + b2c2 + c2a2).
Dễ thấy này là hệ quả của hai bất đẳng thức sau đây
ab(a2 + b2) + ca(c2 + a2) + ca(c2 + a2) > 2(a2b2 + b2c2 + c2a2), (2.7.2)
a4 + b4 + c4 + abc(a+ b+ c) > ab(a2 + b2) + ca(c2 + a2) + ca(c2 + a2). (2.7.3)
Bất đẳng thức (2.7.2) tương đương với
ab(a− b)2 + bc(b− c)2 + ca(c− a)2 > 0.
Ta chứng minh (2.7.3), chuẩn hóa cho a+b+c = 3 và đặt a = x+1, b = y+1, c = −x−y+1
với xy > 0 rồi thế vào bất đẳng thức trên và thu gọn lại, ta được
4(x2 + xy + y2)2 + 3(x2 + xy + y2) > 18xy(x+ y).
Theo bất đẳng thức AM-GM ta có
4(x2 + xy + y2)2 + 3(x2 + xy + y2) > 4 · (3xy)2 + 3 · 3
4
(x+ y)2
= 36x2y2 +
9
4
(x+ y)2,
và
36x2y2 +
9
4
(x+ y)2 − 18xy(x+ y) = 9(x+ y − 4xy)
2
4
> 0.
Đẳng thức xảy ra khi a = b = c hoặc a = b, c = 0. Như vậy bài toán được chứng minh.
Bài 2.8. Cho ba số thực a, b, c thỏa mãn abc = −1, chứng minh rằng
a4 + b4 + c4 + 3(a+ b+ c) > a
2 + b2
c
+
b2 + c2
a
+
c2 + a2
b
.
(Iran MO 2005)
8
Một số vấn đề về bất đẳng thức bậc bốn ba biến
Lời giải. Ta viết bất đẳng thức lại dưới dạng thuần nhất như sau
a4 + b4 + c4 − 3abc(a+ b+ c) > −ab(a2 + b2)− bc(b2 + b2)− ca(c2 + a2). (2.8.1)
Vì ∑
ab(a2 + b2) = (ab+ bc+ ca)(a2 + b2 + c2)− abc(a+ b+ c),
nên (2.8.1) tương đương với
a4 + b4 + c4 + (ab+ bc+ ca)(a2 + b2 + c2) > 4abc(a+ b+ c). (2.8.2)
Nếu a+ b+ c = 0, thay c = −a− b vào (2.8.2) ta được
a4 + b4 + (a+ b)4 − (a2 + ab+ b2) [a2 + b2 + (a+ b)2] > 0,
tuy nhiên dễ thấy
a4 + b4 + (a+ b)4 − (a2 + ab+ b2) [a2 + b2 + (a+ b)2] = 0,
nên bài toán đúng trong trường hợp này.
Nếu a+ b+ c 6= 0, thay (a, b, c) bởi (−a, −b, −c) thì bất đẳng thức vẫn không thay đổi
nên ta có thể giả sử a+ b+ c > 0, và chuẩn hóa cho a+ b+ c = 3.
Đặt a = x+ 1, b = y + 1 thì c = −x− y + 1, rồi thay vào (2.8.2) và thu gọn lại ta được
27(x2 + xy + y2) > 0.
Bất đẳng thức này luôn đúng. Đẳng thức xảy ra khi a = b = c hoặc a + b + c = 0. Bài
toán được chứng minh.
Nhận xét. Bài toán là hệ quả của đẳng thức sau∑
a4 +
∑
bc
∑
a2 = (a+ b+ c)2(a2 + b2 + c2 − ab− bc− ca) + 4abc(a+ b+ c).
Bài 2.9. Chứng minh rằng với mọi số thực a, b, c ta luôn có
(a+ b)4 + (b+ c)4 + (c+ a)4 > 4
7
[
a4 + b4 + c4 + (a+ b+ c)4
]
. (2.9.1)
Lời giải. Nếu a+ b+ c = 0, thay c = −a− b vào (2.9.1) và thu gọn ta được
a4 + 2a3b+ 3a2b2 + 2ab3 + b4 > 0,
ta có
a4 + 2a3b+ 3a2b2 + 2ab3 + b4 = (a2 + ab+ b2)2 > 0.
Nếu a+ b+ c 6= 0, thay (a, b, c) bởi (−a, −b, −c) thì bất đẳng thức vẫn không thay đổi
nên ta có thể giả sử a+ b+ c > 0, và chuẩn hóa cho a+ b+ c = 3.
Đặt a = x+ 1, b = y + 1, c = z + 1, thì x+ y + z = 0 và x3 + y3 + z3 = 3xyz, ta có
(a+ b)4 + (b+ c)4 + (c+ a)4 = x4 + y4 + z4 + 24(x2 + y2 + z2)− 24xyz + 48,
9
Một số vấn đề về bất đẳng thức bậc bốn ba biến
tương tự thì
a4 + b4 + c4 + (a+ b+ c)4 = x4 + y4 + z4 + 6(x2 + y2 + z2) + 12xyz + 84.
Như vậy bất đẳng thức trở thành
x4 + y4 + z4 + 48(x2 + y2 + z2) > 72xyz. (2.9.2)
Giả sử xy > 0 rồi thay z = −x− y vào (2.9.2), bất đẳng thức được viết lại dưới dạng
2(x2 + xy + y2)2 + 96(x2 + xy + y2) + 72xy(x+ y) > 0.
Theo bất đẳng thức AM-GM, ta có
2(x2 + xy + y2)2 + 96(x2 + xy + y2) > 18x2y2 + 72(x+ y)2,
lại có
18x2y2 + 72(x+ y)2 + 72xy(x+ y) = 18(2x+ 2y + xy)2 > 0.
Đẳng thức xảy ra khi và chỉ khi a = b = c hoặc −3a = −3b = c. Chứng minh của chúng
ta vì thế hoàn tất.
Nhận xét. Bài toán là một kết quả mạnh hơn của bất đẳng thức sau
(a+ b)4 + (b+ c)4 + (c+ a)4 > 4
7
(a4 + b4 + c4).
(Việt Nam TST 1996)
Bài 2.10. Với ba số thực a, b, c. Chứng minh rằng với mọi số thực k ta luôn có∑
(a− b)(a− c)(a− kb)(a− kc) > 0. (2.10.1)
Lời giải. Lập luận như trên giả sử a + b + c > 0, và chuẩn hóa cho a + b + c = 3. Đặt
a = x+ 1, b = y + 1, c = −x− y + 1 với xy > 0, khi đó bất đẳng thức (2.10.1) được viết
lại dưới dạng
(k + 2)2(x2 + xy + y2)2 + 3(k − 1)2(x2 + xy + y2) > 9(k + 2)(1− k)xy(x+ y). (2.10.2)
Theo bất đẳng thức AM-GM, ta có
x2 + xy + y2 > 3
4
(x+ y)2 > 3xy > 0.
Suy ra
(k + 2)2(x2 + xy + y2)2 + 3(k − 1)2(x2 + xy + y2) > 9(k + 2)2x2y2 + 9
4
(k − 1)2(x+ y)2,
Ta sẽ chứng minh
(k + 2)2x2y2 +
(k − 1)2(x+ y)2
4
> (k + 2)(1− k)xy(x+ y),
tương đương với
[(k − 1)(x+ y) + 2(k + 2)xy]2 > 0.
Đẳng thức xảy a = b = c hoặc a = kb = kc. Chứng minh hoàn tất.
10
Một số vấn đề về bất đẳng thức bậc bốn ba biến
Nhận xét. Bài toán là hệ quả của đẳng thức
2
∑
(a− b)(a− c)(a− kb)(a− kc) =
∑
(a− b)2 [a+ b− (k + 1)c]2 .
Đây là một kết quả thú vị, chúng ta có một số kết quả khá đẹp mắt như sau
Nếu k = 0, bất đẳng thức trở thành
a2(a− b)(a− c) + b2(b− c)(b− a) + c2(c− a)(c− b) > 0,
đây chính là bất đẳng thức Schur bậc 4 và nếu khai triển ta sẽ được bất đẳng thức (2.7.3).
Nếu k = 2, bất đẳng thức trở thành
9(a4 + b4 + c4) + 126(a2b2 + b2c2 + c2a2) > 5(a+ b+ c)4.
Đẳng thức xảy ra khi a2 = b = c.
3. Đưa bài toán về dạng bậc bốn
Có rất nhiều bài toán dạng phân thức, căn thức, ... thoạt nhìn sẽ không có dạng đa thức
bậc 4 nhưng nếu thông qua một số đánh giá và biến đổi, ta có thể chuyển chúng về dạng
đa thức bậc 4 và có thể áp dụng phương pháp trên. Để hiểu rõ hơn, mời ban đọc xét qua
các bài toán sau.
Bài 3.1. Cho a, b, c là các số thực không âm thỏa mãn ab + bc + ca > 0. Chứng minh
rằng bất đẳng thức sau luôn đúng
(a+ b)2
c2 + ab
+
(b+ c)2
a2 + bc
+
(c+ a)2
b2 + ca
> 6.
Lời giải. Áp dụng bất đẳng thức Cauchy-Schwarz ta có∑ (a+ b)2
c2 + ab
> [(a+ b)
2 + (b+ c)2 + (c+ a)2]2
(ab+ c2)(a+ b)2 + (bc+ a2)(b+ c)2 + (ca+ b2)(c+ a+ b)2
=
4(a2 + b2 + c2 + ab+ bc+ ca)2
(ab+ c2)(a+ b)2 + (bc+ a2)(b+ c)2 + (ca+ b2)(c+ a+ b)2
.
Vì thế để chứng minh bài toán ta cần chứng minh được
(a2 + b2 + c2 + ab+ bc+ ca)2
(ab+ c2)(a+ b)2 + (bc+ a2)(b+ c)2 + (ca+ b2)(c+ a)2
> 3
2
.
Bằng cách khai triển trực tiếp ta thấy điều này tương đương với
2[a4 + b4 + c4 + abc(a+ b+ c)−
∑
ab(a2 + b2)] + 3
∑
ab(a− b)2 > 0.
Bất đẳng thức cuối cùng đúng theo bất đẳng thức (2.7.3), đẳng thức xảy ra khi và chỉ
khi a = b = c hoặc a = b, c = 0 và các hoán vị. Bài toán được chứng minh.
Bài 3.2. Cho a, b, c là các số thực không âm thỏa mãn a2 + b2 + c2 + ab+ bc+ ca = 6.
Chứng minh rằng
1
4− ab +
1
4− bc +
1
4− ca 6 1 (3.2.1).
11
Một số vấn đề về bất đẳng thức bậc bốn ba biến
Lời giải. Bài toán này được làm chặt từ đề thi Moldova TST 2005 và có đến hai dấu bằng
là a = b = c = 1 và a = b =
√
2, c = 0. Muốn sử dụng bất đẳng thức Cauchy-Schwarz ta
phải dùng đến kỹ thuật thêm bớt để làm đảo chiều bài toán. Ta xét
P (a, b) = k − 1
4− ab.
Ta có
P (1, 1) = k − 1
3
,
P
(√
2, 0
)
= k − 1
4
,
P
(√
2,
√
2
)
= k − 1
2
.
Ta cần tìm k sao cho P (a, b) > 0 và đánh giá này càng chặt càng tốt từ các tính toán
trên ta thấy k > 1
2
là giá trị tốt nhất cần tìm, từ đó dẫn đến đẳng thức
1− 2
4− ab =
2− ab
4− ab,
bất đẳng thức (3.2.1) được viết lại như sau
2− ab
4− ab +
2− bc
4− bc +
2− bc
4− bc > 1.
Do a2 + b2 + c2 + ab+ bc+ ca = 6, nên
3(2− ab) = (a− b)2 + c2 + bc+ ca > 0.
Theo bất đẳng thức Cauchy-Schwarz, ta có
∑ 2− ab
4− ab >
[
∑
(2− ab)]2∑
(2− ab)(4− ab) =
(6− ab− bc− ca)2∑
(2− ab)(4− ab) =
(a2 + b2 + c2)2∑
(2− ab)(4− ab) .
Như vậy, ta cần chứng minh
(a2 + b2 + c2)2 >
∑
(2− ab)(4− ab),
tương đương với
a4 + b4 + c4 + a2b2 + b2c2 + c2a2 + 6(ab+ bc+ ca) > 24,
hay dưới dạng thuần nhất∑
a4 +
∑
a2b2 +
(∑
ab
)(∑
a2 +
∑
bc
)
> 2
3
(∑
a2 +
∑
bc
)2
.
Khi triển và thu gọn lại ta được
a4 + b4 + c4 + abc(a+ b+ c) > ab(a2 + b2) + bc(b2 + c2) + ca(c2 + a2).
Đây chính là bất đẳng thức (2.7.3). Bài toán được chứng minh.
12
Một số vấn đề về bất đẳng thức bậc bốn ba biến
Bài 3.3. Cho ba số thực x, y, z thuộc [−1, 1] thỏa mãn x+ y+ z = 0. Chứng minh rằng√
1 + x+ y2 +
√
1 + y + z2 +
√
1 + z + x2 > 3.
Lời giải. Bình phương hai vế, ta được
x2 + y2 + z2 + 2
∑√
(1 + x+ y2)(1 + y + z2) > 6.
Áp dụng bất đẳng thức Cauchy-Schwarz ta có∑√
(1 + x+ y2)(1 + y + z2) >
∑√
(1 + x)(1 + y) +
∑
|yz|
>
∑√
(1 + x)(1 + y)−
∑
yz.
Ta đưa bài toán về chứng minh∑
x2 − 2
∑
yz + 2
∑√
(1 + x)(1 + y) > 6.
Đặt 1+x = a2, 1+y = b2, 1+z = c2 với a, b, c là các số không âm, khi đó a2+b2+c2 = 3,
bất đẳng thức trên trở thành∑
a4 − 2
∑
b2c2 + 2
∑
a2 + 2
∑
bc− 9 > 0,
tương đương với ∑
a4 − 2
∑
b2c2 + 2
∑
bc−
∑
a2 > 0,
3
∑
a4 − 6
∑
b2c2 +
(
2
∑
bc−
∑
a2
)(∑
a2
)
> 0,
khai triển và thu gọn lại, ta được
a4 + b4 + c4 + abc(a+ b+ c)−
∑
ab(a2 + b2) + 2
∑
bc(b− c)2 > 0.
Bất đẳng thức này đúng theo bất đẳng thức (2.7.3), đẳng thức xảy ra khi x = y = z = 0.
Bài toán được chứng minh.
Bài 3.4. Cho ba số thực không âm a, b, c thỏa mãn a2 + b2 + c2 > 0. Chứng minh rằng
a2 + b2 + c2 +
√
3 ·
3
√
abc(ab+ bc

Tài liệu đính kèm:

  • pdfBat_dang_thuc_bac_bon_ba_bien.pdf